Is sum of the reciprocal of any subsequence of natural number divergent?












-1












$begingroup$


Let, ${a_n}$ be any subsequence of natural numbers.



Then, is $sumfrac{1}{a_n}$ always divergent?










share|cite|improve this question











$endgroup$








  • 2




    $begingroup$
    This is almost like asking if there are any series that converge...
    $endgroup$
    – Clayton
    Dec 6 '18 at 5:33
















-1












$begingroup$


Let, ${a_n}$ be any subsequence of natural numbers.



Then, is $sumfrac{1}{a_n}$ always divergent?










share|cite|improve this question











$endgroup$








  • 2




    $begingroup$
    This is almost like asking if there are any series that converge...
    $endgroup$
    – Clayton
    Dec 6 '18 at 5:33














-1












-1








-1


1



$begingroup$


Let, ${a_n}$ be any subsequence of natural numbers.



Then, is $sumfrac{1}{a_n}$ always divergent?










share|cite|improve this question











$endgroup$




Let, ${a_n}$ be any subsequence of natural numbers.



Then, is $sumfrac{1}{a_n}$ always divergent?







sequences-and-series






share|cite|improve this question















share|cite|improve this question













share|cite|improve this question




share|cite|improve this question








edited Dec 6 '18 at 5:34







Tom.

















asked Dec 6 '18 at 5:31









Tom.Tom.

14218




14218








  • 2




    $begingroup$
    This is almost like asking if there are any series that converge...
    $endgroup$
    – Clayton
    Dec 6 '18 at 5:33














  • 2




    $begingroup$
    This is almost like asking if there are any series that converge...
    $endgroup$
    – Clayton
    Dec 6 '18 at 5:33








2




2




$begingroup$
This is almost like asking if there are any series that converge...
$endgroup$
– Clayton
Dec 6 '18 at 5:33




$begingroup$
This is almost like asking if there are any series that converge...
$endgroup$
– Clayton
Dec 6 '18 at 5:33










5 Answers
5






active

oldest

votes


















1












$begingroup$

No. Consider the subsequence defined by the powers of two: ${2,4,8,16,32,64,...}$. This results in the summation



$$sum_{k=1}^infty frac{1}{2^k} = frac{1}{2} +frac{1}{4} +frac{1}{8} +frac{1}{16} +frac{1}{32} + ...$$



We note that this is an infinite geometric series of ratio $1/2$. Thus, from the formula for the summation of such a series, it can be shown that this summation converges to $1$.



Truly many such series exist; this is just one such example.






share|cite|improve this answer









$endgroup$





















    1












    $begingroup$

    No. It is easy to produce a counter example



    Let $a_n=n^2$. So it is the sequence of square numbers 1, 4, 9 ,16 ,..



    Then it is known that:
    $$sum_{n=1}^infty frac 1{n^2} = frac{pi^2}{6}$$






    share|cite|improve this answer









    $endgroup$





















      1












      $begingroup$

      $sum_{k=1}^{infty}frac{1}{k^p}:left{begin{matrix}
      mathrm{converges} &,p>1 \
      mathrm{diverges} &,pleq1
      end{matrix}right.$






      share|cite|improve this answer









      $endgroup$





















        1












        $begingroup$

        No. Even though $sum 1/n$ diverges,
        if you discard all the terms whose denominator involves the digit $9$, the series would become convergent, and the sum would be less than $80$.






        share|cite|improve this answer









        $endgroup$













        • $begingroup$
          Several correct answers to this question, but this is the only cute answer (so far).
          $endgroup$
          – Jyrki Lahtonen
          Dec 6 '18 at 10:15



















        0












        $begingroup$

        Let $n$ be any natural number greater than $1$,



        $$sum_{k=0}^inftyfrac{1}{n^k}=frac{1}{1-frac1n}=frac{n}{n-1}$$



        is a geometric series that converges.






        share|cite|improve this answer









        $endgroup$













          Your Answer





          StackExchange.ifUsing("editor", function () {
          return StackExchange.using("mathjaxEditing", function () {
          StackExchange.MarkdownEditor.creationCallbacks.add(function (editor, postfix) {
          StackExchange.mathjaxEditing.prepareWmdForMathJax(editor, postfix, [["$", "$"], ["\\(","\\)"]]);
          });
          });
          }, "mathjax-editing");

          StackExchange.ready(function() {
          var channelOptions = {
          tags: "".split(" "),
          id: "69"
          };
          initTagRenderer("".split(" "), "".split(" "), channelOptions);

          StackExchange.using("externalEditor", function() {
          // Have to fire editor after snippets, if snippets enabled
          if (StackExchange.settings.snippets.snippetsEnabled) {
          StackExchange.using("snippets", function() {
          createEditor();
          });
          }
          else {
          createEditor();
          }
          });

          function createEditor() {
          StackExchange.prepareEditor({
          heartbeatType: 'answer',
          autoActivateHeartbeat: false,
          convertImagesToLinks: true,
          noModals: true,
          showLowRepImageUploadWarning: true,
          reputationToPostImages: 10,
          bindNavPrevention: true,
          postfix: "",
          imageUploader: {
          brandingHtml: "Powered by u003ca class="icon-imgur-white" href="https://imgur.com/"u003eu003c/au003e",
          contentPolicyHtml: "User contributions licensed under u003ca href="https://creativecommons.org/licenses/by-sa/3.0/"u003ecc by-sa 3.0 with attribution requiredu003c/au003e u003ca href="https://stackoverflow.com/legal/content-policy"u003e(content policy)u003c/au003e",
          allowUrls: true
          },
          noCode: true, onDemand: true,
          discardSelector: ".discard-answer"
          ,immediatelyShowMarkdownHelp:true
          });


          }
          });














          draft saved

          draft discarded


















          StackExchange.ready(
          function () {
          StackExchange.openid.initPostLogin('.new-post-login', 'https%3a%2f%2fmath.stackexchange.com%2fquestions%2f3028073%2fis-sum-of-the-reciprocal-of-any-subsequence-of-natural-number-divergent%23new-answer', 'question_page');
          }
          );

          Post as a guest















          Required, but never shown

























          5 Answers
          5






          active

          oldest

          votes








          5 Answers
          5






          active

          oldest

          votes









          active

          oldest

          votes






          active

          oldest

          votes









          1












          $begingroup$

          No. Consider the subsequence defined by the powers of two: ${2,4,8,16,32,64,...}$. This results in the summation



          $$sum_{k=1}^infty frac{1}{2^k} = frac{1}{2} +frac{1}{4} +frac{1}{8} +frac{1}{16} +frac{1}{32} + ...$$



          We note that this is an infinite geometric series of ratio $1/2$. Thus, from the formula for the summation of such a series, it can be shown that this summation converges to $1$.



          Truly many such series exist; this is just one such example.






          share|cite|improve this answer









          $endgroup$


















            1












            $begingroup$

            No. Consider the subsequence defined by the powers of two: ${2,4,8,16,32,64,...}$. This results in the summation



            $$sum_{k=1}^infty frac{1}{2^k} = frac{1}{2} +frac{1}{4} +frac{1}{8} +frac{1}{16} +frac{1}{32} + ...$$



            We note that this is an infinite geometric series of ratio $1/2$. Thus, from the formula for the summation of such a series, it can be shown that this summation converges to $1$.



            Truly many such series exist; this is just one such example.






            share|cite|improve this answer









            $endgroup$
















              1












              1








              1





              $begingroup$

              No. Consider the subsequence defined by the powers of two: ${2,4,8,16,32,64,...}$. This results in the summation



              $$sum_{k=1}^infty frac{1}{2^k} = frac{1}{2} +frac{1}{4} +frac{1}{8} +frac{1}{16} +frac{1}{32} + ...$$



              We note that this is an infinite geometric series of ratio $1/2$. Thus, from the formula for the summation of such a series, it can be shown that this summation converges to $1$.



              Truly many such series exist; this is just one such example.






              share|cite|improve this answer









              $endgroup$



              No. Consider the subsequence defined by the powers of two: ${2,4,8,16,32,64,...}$. This results in the summation



              $$sum_{k=1}^infty frac{1}{2^k} = frac{1}{2} +frac{1}{4} +frac{1}{8} +frac{1}{16} +frac{1}{32} + ...$$



              We note that this is an infinite geometric series of ratio $1/2$. Thus, from the formula for the summation of such a series, it can be shown that this summation converges to $1$.



              Truly many such series exist; this is just one such example.







              share|cite|improve this answer












              share|cite|improve this answer



              share|cite|improve this answer










              answered Dec 6 '18 at 5:34









              Eevee TrainerEevee Trainer

              5,4491936




              5,4491936























                  1












                  $begingroup$

                  No. It is easy to produce a counter example



                  Let $a_n=n^2$. So it is the sequence of square numbers 1, 4, 9 ,16 ,..



                  Then it is known that:
                  $$sum_{n=1}^infty frac 1{n^2} = frac{pi^2}{6}$$






                  share|cite|improve this answer









                  $endgroup$


















                    1












                    $begingroup$

                    No. It is easy to produce a counter example



                    Let $a_n=n^2$. So it is the sequence of square numbers 1, 4, 9 ,16 ,..



                    Then it is known that:
                    $$sum_{n=1}^infty frac 1{n^2} = frac{pi^2}{6}$$






                    share|cite|improve this answer









                    $endgroup$
















                      1












                      1








                      1





                      $begingroup$

                      No. It is easy to produce a counter example



                      Let $a_n=n^2$. So it is the sequence of square numbers 1, 4, 9 ,16 ,..



                      Then it is known that:
                      $$sum_{n=1}^infty frac 1{n^2} = frac{pi^2}{6}$$






                      share|cite|improve this answer









                      $endgroup$



                      No. It is easy to produce a counter example



                      Let $a_n=n^2$. So it is the sequence of square numbers 1, 4, 9 ,16 ,..



                      Then it is known that:
                      $$sum_{n=1}^infty frac 1{n^2} = frac{pi^2}{6}$$







                      share|cite|improve this answer












                      share|cite|improve this answer



                      share|cite|improve this answer










                      answered Dec 6 '18 at 5:35









                      M.AM.A

                      1599




                      1599























                          1












                          $begingroup$

                          $sum_{k=1}^{infty}frac{1}{k^p}:left{begin{matrix}
                          mathrm{converges} &,p>1 \
                          mathrm{diverges} &,pleq1
                          end{matrix}right.$






                          share|cite|improve this answer









                          $endgroup$


















                            1












                            $begingroup$

                            $sum_{k=1}^{infty}frac{1}{k^p}:left{begin{matrix}
                            mathrm{converges} &,p>1 \
                            mathrm{diverges} &,pleq1
                            end{matrix}right.$






                            share|cite|improve this answer









                            $endgroup$
















                              1












                              1








                              1





                              $begingroup$

                              $sum_{k=1}^{infty}frac{1}{k^p}:left{begin{matrix}
                              mathrm{converges} &,p>1 \
                              mathrm{diverges} &,pleq1
                              end{matrix}right.$






                              share|cite|improve this answer









                              $endgroup$



                              $sum_{k=1}^{infty}frac{1}{k^p}:left{begin{matrix}
                              mathrm{converges} &,p>1 \
                              mathrm{diverges} &,pleq1
                              end{matrix}right.$







                              share|cite|improve this answer












                              share|cite|improve this answer



                              share|cite|improve this answer










                              answered Dec 6 '18 at 5:47









                              Hussain-AlqatariHussain-Alqatari

                              3187




                              3187























                                  1












                                  $begingroup$

                                  No. Even though $sum 1/n$ diverges,
                                  if you discard all the terms whose denominator involves the digit $9$, the series would become convergent, and the sum would be less than $80$.






                                  share|cite|improve this answer









                                  $endgroup$













                                  • $begingroup$
                                    Several correct answers to this question, but this is the only cute answer (so far).
                                    $endgroup$
                                    – Jyrki Lahtonen
                                    Dec 6 '18 at 10:15
















                                  1












                                  $begingroup$

                                  No. Even though $sum 1/n$ diverges,
                                  if you discard all the terms whose denominator involves the digit $9$, the series would become convergent, and the sum would be less than $80$.






                                  share|cite|improve this answer









                                  $endgroup$













                                  • $begingroup$
                                    Several correct answers to this question, but this is the only cute answer (so far).
                                    $endgroup$
                                    – Jyrki Lahtonen
                                    Dec 6 '18 at 10:15














                                  1












                                  1








                                  1





                                  $begingroup$

                                  No. Even though $sum 1/n$ diverges,
                                  if you discard all the terms whose denominator involves the digit $9$, the series would become convergent, and the sum would be less than $80$.






                                  share|cite|improve this answer









                                  $endgroup$



                                  No. Even though $sum 1/n$ diverges,
                                  if you discard all the terms whose denominator involves the digit $9$, the series would become convergent, and the sum would be less than $80$.







                                  share|cite|improve this answer












                                  share|cite|improve this answer



                                  share|cite|improve this answer










                                  answered Dec 6 '18 at 5:47









                                  xbhxbh

                                  6,0231522




                                  6,0231522












                                  • $begingroup$
                                    Several correct answers to this question, but this is the only cute answer (so far).
                                    $endgroup$
                                    – Jyrki Lahtonen
                                    Dec 6 '18 at 10:15


















                                  • $begingroup$
                                    Several correct answers to this question, but this is the only cute answer (so far).
                                    $endgroup$
                                    – Jyrki Lahtonen
                                    Dec 6 '18 at 10:15
















                                  $begingroup$
                                  Several correct answers to this question, but this is the only cute answer (so far).
                                  $endgroup$
                                  – Jyrki Lahtonen
                                  Dec 6 '18 at 10:15




                                  $begingroup$
                                  Several correct answers to this question, but this is the only cute answer (so far).
                                  $endgroup$
                                  – Jyrki Lahtonen
                                  Dec 6 '18 at 10:15











                                  0












                                  $begingroup$

                                  Let $n$ be any natural number greater than $1$,



                                  $$sum_{k=0}^inftyfrac{1}{n^k}=frac{1}{1-frac1n}=frac{n}{n-1}$$



                                  is a geometric series that converges.






                                  share|cite|improve this answer









                                  $endgroup$


















                                    0












                                    $begingroup$

                                    Let $n$ be any natural number greater than $1$,



                                    $$sum_{k=0}^inftyfrac{1}{n^k}=frac{1}{1-frac1n}=frac{n}{n-1}$$



                                    is a geometric series that converges.






                                    share|cite|improve this answer









                                    $endgroup$
















                                      0












                                      0








                                      0





                                      $begingroup$

                                      Let $n$ be any natural number greater than $1$,



                                      $$sum_{k=0}^inftyfrac{1}{n^k}=frac{1}{1-frac1n}=frac{n}{n-1}$$



                                      is a geometric series that converges.






                                      share|cite|improve this answer









                                      $endgroup$



                                      Let $n$ be any natural number greater than $1$,



                                      $$sum_{k=0}^inftyfrac{1}{n^k}=frac{1}{1-frac1n}=frac{n}{n-1}$$



                                      is a geometric series that converges.







                                      share|cite|improve this answer












                                      share|cite|improve this answer



                                      share|cite|improve this answer










                                      answered Dec 6 '18 at 7:49









                                      Siong Thye GohSiong Thye Goh

                                      100k1465117




                                      100k1465117






























                                          draft saved

                                          draft discarded




















































                                          Thanks for contributing an answer to Mathematics Stack Exchange!


                                          • Please be sure to answer the question. Provide details and share your research!

                                          But avoid



                                          • Asking for help, clarification, or responding to other answers.

                                          • Making statements based on opinion; back them up with references or personal experience.


                                          Use MathJax to format equations. MathJax reference.


                                          To learn more, see our tips on writing great answers.




                                          draft saved


                                          draft discarded














                                          StackExchange.ready(
                                          function () {
                                          StackExchange.openid.initPostLogin('.new-post-login', 'https%3a%2f%2fmath.stackexchange.com%2fquestions%2f3028073%2fis-sum-of-the-reciprocal-of-any-subsequence-of-natural-number-divergent%23new-answer', 'question_page');
                                          }
                                          );

                                          Post as a guest















                                          Required, but never shown





















































                                          Required, but never shown














                                          Required, but never shown












                                          Required, but never shown







                                          Required, but never shown

































                                          Required, but never shown














                                          Required, but never shown












                                          Required, but never shown







                                          Required, but never shown







                                          Popular posts from this blog

                                          To store a contact into the json file from server.js file using a class in NodeJS

                                          Redirect URL with Chrome Remote Debugging Android Devices

                                          Dieringhausen